2025 AMC 10A Problems/Problem 9
Let
. For how many real numbers
does the graph of
pass through the point
?
(A)
(B)
(C)
(D)
(E) more than
Video Solution
Solution 1
Substitute
for
and set this expression equal to
The problem boils down to finding how many real roots
has. We further simplify this expression and create a function
Using Descarte's Rule of Signs we get:
Sign changes for
(possible number of positive roots): 2
Sign changes for
(possible number of negative roots): 1
Possibilities for roots:
1)
positive roots,
negative root
2)
positive roots,
negative root,
imaginary roots
So which one is it? We know if the function changes sign between an interval, then a root exists in that interval. From
to
the function changes sign because
while
, so a positive root exists. This eliminates the second possibility, implying that there must be
positive and
negative roots. So the answer is
Alternative Solution
Since we are trying to find the number of roots of
(which is the same as the number of roots of
because we divided by a constant, namely
), we can sketch a graph of this function, which we will call
. We find that the function is strictly decreasing when
,
, and the function is strictly increasing when
. We immediately see that since
is negative, there is a root between
and
by the Intermediate Value Theorem. From here, we use the first derivative test as follows. Taking the derivative, we have
. We solve for the roots of this equation, obtaining
. We know that at these two values, the function will either reach a local minimum or maximum if the second derivative isn't
. Proceeding by the second derivative rule, we have that
is positive at
and negative at
, making them local minimums and local maximums, respectively. We don't need to care about the latter case because the local maximum doesn't give any more roots besides the one we've already found. Considering the first case when
, we must find the sign of
. Using some approximations, we have that
By the Intermediate Value Theorem again, we see that there is one root between
and
and one root between
and
. Thus, we have a total of
roots. Therefore, the answer is
~scjh999999
~minor LaTeX edits by i_am_not_suk_at_math
Solution 2
This problem is essentially asking how many values of
satisfy
since the graph
is a shift
units right from the original graph of
Now we just need to determine the amount of real solutions to
Dividing all terms by 25, we get
Using the rational root theorem and testing, we find no rational roots. Our next method to determine the amount of roots is graphing. Notice how when
When
Substituting
gives us
Substituting
gives us
also. This makes us think to see if there was any sudden bump or curve in the graph, so we test the midpoint which is
Substituting
we get
Aha! Connecting the dots, we see there are two solutions from
Also,
must continue increasing after
which tells us there is a third root when
Now we can conclude that the problem has 3 real roots, meaning that 3 values of a satisfy the problem. The answer is
~ eqb5000/Esteban Q.
Solution 3 (Graphing)
We can factor
as
. We just need to check that if the local maximum between
and
is below or above 25. Observe that
, so the graph will cross the line
3 times,
~gimmeaworkingusername
Somewhat cheese (not recommended)
Instead of plugging in a value between
and
we can assume that the equation will reach a value above
between those numbers. Thus, the graph crosses
3 times,
~dodobird150
Super Lazy Solution
Instead of using numbers, we could just remember the shape of the graph given, and that it is extremely vertically dilated. The shape of the graph is(for the y-values given) basically
lines, so moving these
lines left and right, we can see that the graph will intersect at 3 points, so
~HappyLion
Solution 4 (Discriminant)
We start by writing
Then, we substitute
for
, and substitute
Now, to make this easier, we define
, and find how many real values of
satisfy this
We use the discriminant of a cubic equation, where, assuming the form
,
When
, there are
distinct real solutions, when
, there are repeated solutions (which all belong to
), and when
, there is one real solution and 2 complex-conjugate solutions.
From the expression we derived, we can see that
,
,
, and
We plug this into the discriminant formula, yielding
Therefore,
, meaning there are three real values of
that satisfy the equation. Since
, we know that there also are 3 real values of
satisfying the equation. Thus,
~shockfront99
Chinese Video Solution
https://www.bilibili.com/video/BV1bj2uBxEXo/
~metrixgo
Video Solution by SpreadTheMathLove
https://youtu.be/dAeyV60Hu5c?t=1229
Video Solution 2 graphig approach by SpreadTheMathLove
https://www.youtube.com/watch?v=tDclC-ojfuY
Video Solution (In 1 Min)
https://youtu.be/zvB7LuEBbww?si=OJ9H-Wkkx3WUxTU5 ~ Pi Academy
Video Solution
https://youtu.be/gWSZeCKrOfU?si=MPPNaoCB_Bqo918z&t=1132 ~MK
Video Solution by Daily Dose of Math
https://youtu.be/gPh9w3X3QSw?si=FhsZ3VSzFXKEO5pf&t=749 ~Thesmartgreekmathdude
See Also
| 2025 AMC 10A (Problems • Answer Key • Resources) | ||
| Preceded by Problem 8 |
Followed by Problem 10 | |
| 1 • 2 • 3 • 4 • 5 • 6 • 7 • 8 • 9 • 10 • 11 • 12 • 13 • 14 • 15 • 16 • 17 • 18 • 19 • 20 • 21 • 22 • 23 • 24 • 25 | ||
| All AMC 10 Problems and Solutions | ||
These problems are copyrighted © by the Mathematical Association of America.